Question

The (total) cost function is given by C = 50 + 60Q – 18Q2 + 2Q3...

The (total) cost function is given by C = 50 + 60Q – 18Q2 + 2Q3

e. Write down the average variable cost function AVC(Q). (1) f. Write down the average total cost function AC(Q). (1) g. Find the break-even point (Q and AC). (1) h. Find the shut-down point (Q and AVC). (1) i. Draw a graph to illustrate AC, AVC, and MC functions for quantities Q on the interval between 1 and 10. Make sure you show (put the numbers there) where exactly the MC curve intercepts AVC and AC curves. (1) j. If the price is P = 60, calculate the profit-maximizing firm’s profit. (1)

0 0
Add a comment Improve this question Transcribed image text
Answer #1

Cost function is C = 50 + 60Q – 18Q2 + 2Q3

MC = derivative of C with respect to Q = 60 - 36Q + 6Q^2

AC = C/Q = 50/Q + 60 - 18Q + 2Q^2

AVC = VC/Q = 60 - 18Q + 2Q^2

At break even point AC = MC

60 - 36Q + 6Q^2 = 50/Q + 60 - 18Q + 2Q^2

50/Q + 18Q - 4Q^2 = 0

4Q^3 - 18Q^2 - 50 = 0

This gives Q = 5

AC = 50/5 + 60 - 18*5 + 2*5^2 = 30

Hence break even quantity is 5 and AC = $30

At shut down point AVC = MC

60 - 18Q + 2Q^2 = 60 - 36Q + 6Q^2

18Q - 4Q^2 = 0

Q = 4.5

AVC = 60 - 18*4.5 + 2*4.5^2 = $19.5

Shut down Q is 4.5 units and AVC = $19.5

When P = MC = 60 we have

60 = 60 - 36Q + 6Q^2

Q = 6 units

AC = 50/6 + 60 - 18*6 + 2*6^2 = 32.33

Profit = (P - AC)*Q = (60 - 32.33)*6 = $166

Add a comment
Know the answer?
Add Answer to:
The (total) cost function is given by C = 50 + 60Q – 18Q2 + 2Q3...
Your Answer:

Post as a guest

Your Name:

What's your source?

Earn Coins

Coins can be redeemed for fabulous gifts.

Not the answer you're looking for? Ask your own homework help question. Our experts will answer your question WITHIN MINUTES for Free.
Similar Homework Help Questions
  • Consider the cost function C(Q) = 60Q – 12Q2 + Q3. a.   Sketch the total cost,...

    Consider the cost function C(Q) = 60Q – 12Q2 + Q3. a.   Sketch the total cost, the total variable cost, and the fixed cost for 0-12 units of output. b.   Find expressions for the ATC, AVC, AFC, and MC curves, and sketch the curves on a single graph, making sure the curves are correctly related to each other. At what output level does ATC reaches its minimum? c.   What can you say about the shape of the marginal product of...

  • 1) A perfectly competitive firm faces the following Total revenue, Total cost and Marginal cost functions:...

    1) A perfectly competitive firm faces the following Total revenue, Total cost and Marginal cost functions: TR = 10Q TC = 2 + 2Q + Q2 MC = 2 + 2Q At the level of output maximizing profit , the above firm's level of economic profit is                                                                                                           A) $0 B) $4 C) $6 D) $8 *Additional information after I did the math: The price this firm charges for its product is $10, the level of output maximizing profit is 4...

  • Suppose a firm has a total cost function, T C = 3/8(Q^2) − 50, and therefore...

    Suppose a firm has a total cost function, T C = 3/8(Q^2) − 50, and therefore marginal costs of MC = 3/4Q. Assume the market for this firm’s goods is perfectly competitive with a market price, P = 24. (a) Given the information above, is the firm in the short-run or long-run? (1 point) (b) Write down the firm’s marginal revenue equation. (1 points) (c) How many units should the firm produce if it wants to maximize profit? (3 points)...

  • 2. (54 points) Short-run costs. Suppose w 1, r 10 and K 20. C )q3 +200 a) (5 points) We have TC = WG) q3 + rK = On one graph (with q on the horizontal axis), graph the Total Cost, Variable Cost, and...

    2. (54 points) Short-run costs. Suppose w 1, r 10 and K 20. C )q3 +200 a) (5 points) We have TC = WG) q3 + rK = On one graph (with q on the horizontal axis), graph the Total Cost, Variable Cost, and Fixed Cost functions. Pay attention to the shape of the curves, where they intercept the axes and each other (if they do), and the position of the curves relative to each other b) (9 points) Using...

  • Consider a competitive rm with total costs given by TC(q) = 100 + 10q + q^2,...

    Consider a competitive rm with total costs given by TC(q) = 100 + 10q + q^2, The firm faces a market price p = 50. (a) Write expressions for total revenue TR and marginal revenue MR as functions of output q. (b) Write expressions for average total cost ATC, average variable cost AVC, and marginal cost MC as functions of output q. (c) For what value of output is ATC minimized? (d) Find the profit maximizing level of output q...

  • 1. A perfectly competitive firm sells its product for $360/unit and has an average total cost...

    1. A perfectly competitive firm sells its product for $360/unit and has an average total cost function given by: ATC(Q) = 1000/Q + 30 + 1.5Q. a. What are this firm’s fixed costs? Explain. b. Determine this firm’s profit maximizing level of output. c. Calculate this firm’s profits. 2. A perfectly competitive firm sells its product for $200/unit and has a total cost of production given by: C(Q) = 1500 + 40Q+5Q2 . a. What are this firm’s fixed costs?...

  • Suppose a firm’s inverse demand curve is given by P=120-.5Q and its cost equation is C=420+60Q+Q2....

    Suppose a firm’s inverse demand curve is given by P=120-.5Q and its cost equation is C=420+60Q+Q2. Find the firm’s optimal quantity, price and profit (1) by using the profit and marginal profit equation and (2) by setting MR equal to MC. Also provide a graph of MR and MC. Suppose instead that the firm can sell any and all of its output at the fixed market price P=120. Find the firm’s optimal output.

  • 2. Suppose the firm has the one variable production function Q=L?. Assume that the wage rate...

    2. Suppose the firm has the one variable production function Q=L?. Assume that the wage rate is w= 20 and that the firm has fixed costs of 10. Finally, assume that the firm is a price taker and the market price is 2. a) Show that this production function exhibits increasing returns to scale. Show that the marginal product of labor is increasing. Illustrate the production function. Is it convex, concave or neither? b) Find the variable and total cost...

  • A firm’s short run cost function is C(q)=150q-4q^2+0.4q^3+275 . Determine the fixed cost, F; the average...

    A firm’s short run cost function is C(q)=150q-4q^2+0.4q^3+275 . Determine the fixed cost, F; the average variable cost, AVC; Average Fixed Cost, AFC; Average Cost, AC; and the Marginal Cost, MC.

  • A firm operates in a perfectly competitive market with a price of P = 50 for...

    A firm operates in a perfectly competitive market with a price of P = 50 for the product. TVC = 0.5Q3 − 18Q2 + 170Q Q (output) TFC = 300. Write an equation expressing the firm’s total revenue (TR) as function of Q. Write an equation expressing the firm’s total cost (TC), as a function of Q. Write an equation expressing the firm’s profit (π), as a function of Q.Find the first-order condition for the firm’s profit-maximization decision. Find the...

ADVERTISEMENT
Free Homework Help App
Download From Google Play
Scan Your Homework
to Get Instant Free Answers
Need Online Homework Help?
Ask a Question
Get Answers For Free
Most questions answered within 3 hours.
ADVERTISEMENT
ADVERTISEMENT
ADVERTISEMENT